Траектория снаряда, брошенного под гору

Я изучаю механику и задался целью решить задачу определения оптимального угла броска снаряда стоя на возвышенности, на максимальную дальность. Мой ответ кажется почти правдоподобным, за исключением одного члена, который, чтобы быть правдоподобным, должен поменять свой знак. Но я не могу найти пробела в своих рассуждениях.

Проблема: я стою на прямом склоне холма и хочу бросить камень с максимальной дальности. Холм наклонен от горизонтали на ф . Какой угол θ выше горизонтали я должен бросить его?

Мое решение:

  1. Используйте координаты, чтобы Икс находится параллельно холму

  2. Позволять α "=" ф + θ (то есть угол над землей, на который я бросаю)

  3. Затем начальная в является в Икс "=" потому что α , в у "=" грех α (нормализация единиц для удаления любых констант)

  4. Тогда ускорение свободного падения равно а Икс "=" к грех ф , а у "=" к потому что ф (гравитация в направлении y). Для упрощения расчетов предположим к "=" 2 (ответ верен для любого значения гравитации, поэтому на Луне он такой же, как и на Земле)

  5. Мы хотим найти альфу, которая максимизирует с Икс в то время, что делает с у "=" 0 . Сначала найдите время, за которое с у "=" 0 ; назовите это т.

  6. с у "=" т грех α т 2 потому что ф . Используя квадратичную формулу, с у "=" 0 в т "=" 0 или т "=" грех α потому что ф .

  7. Теперь найди с Икс в этот т . Подставляя и используя базовую алгебру и триггер, мы получаем с Икс "=" грех α потому что α грех 2 α загар ф . (Это имеет смысл; первый член достигает максимума при π / 4 , как и следовало ожидать от симметрии. Второе слагаемое говорит нам, что если земля значительно проседает, мы должны уменьшить угол бросания. Очень правдоподобно.)

  8. Принимая фи за константу, мы хотим максимизировать это выражение. Небольшое исчисление и триггерные тождества дают производную, равную потому что ( 2 α ) грех ( 2 α ) загар ф , который имеет ноль в α "=" π / 4 ф / 2 , или θ "=" π / 4 3 ф / 2 . Вот где вещи ломаются. Первый срок, π / 4 , кажется правильным. Но второй член дает смехотворные результаты.

  9. Переключение знака второго члена в альфа-уравнении приводит к θ "=" π / 4 ф / 2 , что дает вполне правдоподобные результаты. Но я не могу найти ни одной ошибки в своих рассуждениях или расчетах!

Кто-нибудь может найти недостающую ссылку?


Пояснение к ответу:

Как определил Пигмалион, шаг 4 неверен. а у значение правильное, но , а Икс должен быть положительным : указывая вниз по склону.

Ответ не зависит от величины силы тяжести; но это зависит от направления .

Пересмотр вывода:

7. с Икс "=" грех α потому что α + грех 2 α загар ф

8. Производная потому что ( 2 α ) + грех ( 2 α ) загар ф , с нулем в α "=" π / 4 + ф / 2 , таким образом θ "=" π / 4 ф / 2 . КЭД.

Добро пожаловать в PhysSE. Пожалуйста, в следующий раз используйте математическое форматирование, чтобы x = sin theta стало Икс "=" грех θ например.
Один момент: я утверждаю, что ответ не зависит от гравитации, как на Луне и Марсе. А как насчет «обратной гравитации» (отталкивания от земли)? Это наверняка изменило бы ответ. Это не решает проблемы, но может быть хорошим началом.
Почему бы не сохранить горизонтальную вертикальную систему координат (для Икс , у компоненты) и соответствуют траектории у ( т ) , Икс ( т ) к нисходящему склону загар ф или у ( т ) "=" Икс ( т ) загар ф ?
@S.RobertJames: я думаю, что ответ зависит от отношения начальной скорости к силе тяжести.
Хорошо, это не зависит от г . Моя формула р "=" 2 ты 2 г потому что ф ( потому что θ грех ( ф + θ ) ) , максимум при θ "=" π 2 ф 2 . Что у тебя?
@Manishearth Максимум должен быть π / 4 + что-то. Проверить лимит ф "=" 0
@Pygmalion: Да, это так. π 4 (неправильно скопировал из блокнота), но минус все равно получаю. Вы уверены, что бросаете мяч вниз ? В этом случае есть плюс.
@Manishearth Может быть и так. Вычисление вниз кажется более интуитивной проблемой, чем наоборот. С.РобертДжеймс расскажет нам...

Ответы (1)

Насколько я вижу, а у "=" к потому что ( ф ) < 0 , но а Икс "=" к грех ( ф ) > 0 ! По крайней мере, если вы бросаете снаряд вниз...

Я считаю, что они оба < 0 . а происходит из-за гравитации, а гравитация направлена ​​прямо вниз, вращается по часовой стрелке на ф из-за поворота координат. вращающийся [ 0 1 ] к ф , 0 < ф < π / 2 дает отрицательные значения обоих компонентов.
@S.RobertJames Если вы бросаете вниз , максимальное расстояние больше , это потому, что а у меньше и а Икс положительный (помогает вам). Забудьте о вращении, нарисуйте картинку.
Пожалуйста, объясните, почему вы чувствуете а Икс положительный (помогает). Хотя я интуитивно понимаю вашу точку зрения (гравитация помогает мне сбрасывать с холма), вращение координат (как описано в моем предыдущем комментарии) не дает отрицательного результата (больно) а Икс ?
я читал 0 ! как «нулевой факториал». Заставил меня сказать "подожди, что?". Затем я снова посмотрел на вопрос, чтобы убедиться, что он не рекурсивный (где есть несколько отказов). ;-)
Понял: оси координат вращаются по часовой стрелке. Векторы от начальных координат вращаются против часовой стрелки. Следовательно, а Икс вращается, чтобы быть положительным! Отлично, спасибо Пигмалион. Ты номер е π + 1 ! (снимаю шляпу перед Манишертом).
Да, С.Роберт Джеймс, иногда неплохо обрисовать проблему. :) Если это ответ на ваш вопрос, вы можете принять его, нажав зеленую кнопку под вопросом :)
С. Роберт Джеймс: Чертовски приятно. :)
@Manishearth Я не совсем понял 0! комментарий. Было ли оно направлено на меня?
@Pygmalion Я тоже использовал его в качестве награды :-)
Упс, вместо помощи написал ад. Но я думаю, вы получили правильное сообщение. :)